Bạn chưa đăng nhập. Vui lòng đăng nhập để hỏi bài

Những câu hỏi liên quan
Nguyễn Thị Diệu
Xem chi tiết
Thiên Dương Nam
10 tháng 11 2017 lúc 18:52

đoạn thẳng BN là 10 - 6 = 4 ( cm )

_________BM là 6- 3 = 3 (cm )

_________MN là 4 + 3 = 7 (cm)

k cho mk nha mơn nhìu      ^ __ ^

Hồ Hoài Anh
10 tháng 11 2017 lúc 19:05

độ dài đoạn thẳng AB là

  10-6=4(cm)

độ dài đoạn thẳng BM là

    6-3=3(cm)

độ dài đoạn thẳng MN là

    10-3=7(cm)

           

    

          

           

     

               

Phương Trình Hai Ẩn
Xem chi tiết
duong cao
Xem chi tiết
Tran Thi Hai Lam
15 tháng 4 2018 lúc 20:53

( Bạn tự vẽ hình nhé)

a)

+) Vì M nằm giữa A và B => AM + MB =AB => AM = AB - MB = 3 - 1 = 2 (cm)

+) Vì AM=AN => AN=2cm.

    Vì AN nằm trên tia đối của AB => AM + AN = NM 

                                                     = 2 + 2 = 4 (cm)

+) Vì M nằm giữa A và B nên M nằm giữa N và B => NM + BM = BN

                                                                                = 4 + 1 = 5 (cm)

                     Vậy: BN=5cm.

(Câu b cậu tự làm nha! Mk bận lắm, chỉ giúp đc câu a thôi).

a, ta có: AM + MB= AB

=> AM=AB- MB

hay AM= 3-1

=> AM= 2cm

Mà AM = AN

=> AN= 2cm

ta có: AN+ AB= NB

hay 2+ 3= NB

=> NB= 5 cm

b, khi M nằm cùng điểm B thì BN có độ dài lớn nhất

Khách vãng lai đã xóa
Nguyễn Hương Giang
Xem chi tiết
OniChanPCM
Xem chi tiết
Nguyễn Hương Giang
Xem chi tiết
nguyễn phương linh
Xem chi tiết
Cô nàng Thiên Bình
19 tháng 3 2018 lúc 21:02

a)ta có AM+MN+BN=5

hay AM+1+BN=5

=>AM+BN=5-1

AM+BN=4 cm

nguyễn phương linh
19 tháng 3 2018 lúc 21:06

lm cả hai câu đc ko bn

Ngô Quốc Kỳ
Xem chi tiết
Nguyễn Huy Tú
28 tháng 1 2021 lúc 12:16

A B M N

a, Ta có : AB - MN = 5 - 1 = 4 cm 

hay AM + BN = 4 cm 

b, Ta có : AB - ( AM + BN ) = MN 

<=> 5 - 7 = MN <=> MN = -2 

P/s : viết đề hẳn hoi ý b nhé !!! 

Khách vãng lai đã xóa
Hiền Thảo Bùi
Xem chi tiết
Trần Phương Anh
3 tháng 1 2016 lúc 9:36

Vì M nằm giữa 2 điểm A và B nên khi MB = 1 ta có: MB+MA= 3 mà MB=1 suy ra: MA= 3-1=2(cm)

Theo đề bài ta có : MA= AN suy ra khi MA = 2cm thì AN= 2cm . Ta có : BN = MB + MA+ AN suy ra BN= 1+ 2 +2 = 5(cm)

Để BN có độ dài lớn nhất thì MB phải có độ dài nhỏ nhất